Download as docx, pdf, or txt
Download as docx, pdf, or txt
You are on page 1of 30

Kaitlin McCurdy—Contracts Outline—Grenardo Fall 2022

I. MULTIPLE CHOICE..........................................................................................................................................................................................................................................3
MIDTERM MATERIAL TO KNOW.......................................................................................................................................................................................................................................3
BILATERAL VS. UNILATERAL..................................................................................................................................................................................................................................................8
OPTION CONTRACT..................................................................................................................................................................................................................................................................8
SECTION 27: EXISTENCE OF K WHERE WRITTEN MEMORIAL IS CONTEMPLATED....................................................................................................................................................................8
QUAKE FACTORS:....................................................................................................................................................................................................................................................................8
MAILBOX RULE EXCEPTION....................................................................................................................................................................................................................................................8
CONTRACT FORMATION UNDER UCC.............................................................................................................................................................................................................................8
UCC ARTICLE 2......................................................................................................................................................................................................................................................................8
UCC 1-103B............................................................................................................................................................................................................................................................................8
UCC §2-201 + MERCHANT EXCEPTION..................................................................................................................................................................................................................................8
UCC 2-204: FORMATION IN GENERAL....................................................................................................................................................................................................................................9
UCC 2-205: FIRM OFFER........................................................................................................................................................................................................................................................9
UCC 2-206: OFFER AND ACCEPTANCE IN FORMATION OF K.................................................................................................................................................................................................9
BATTLE OF THE FORMS.......................................................................................................................................................................................................................................................9
PREDOMINATE PURPOSE TEST (COAKLEY).............................................................................................................................................................................................................................9
COMMON LAW: MIRROR IMAGE & LAST SHOT RULES........................................................................................................................................................................................................10
UCC 2-207: ADDITIONAL TERMS IN ACCEPTANCE OR CONFIRMATION (BATTLE OF THE FORMS GUIDE)..........................................................................................................................10
ELECTRONIC CONTRACTING..........................................................................................................................................................................................................................................10
LIABILITY IN THE ABSENCE OF BARGAINED FOR EXCHANGE...........................................................................................................................................................................10
A. PROMISSORY ESTOPPEL.........................................................................................................................................................................................................................................10
SECTION 90: PROMISSORY ESTOPPEL EXISTS WHEN (ALL 4):...................................................................................................................................................................................10
SECTION 87(2): WHEN PROMISSORY ESTOPPEL MAKES THE OFFER IRREVOCABLE..................................................................................................................................................11
B. RESTITUTION..............................................................................................................................................................................................................................................................11
SECTION 86: PROMISSORY RESTITUTION..................................................................................................................................................................................................................11
STATUTE OF FRAUDS..........................................................................................................................................................................................................................................................11
MY LEGS MNEMONIC..........................................................................................................................................................................................................................................................12
R§139: PROMISSORY ESTOPPEL (ON ESSAY TOO) / ENFORCEMENT BY VIRTUE OF ACTION IN RELIANCE..........................................................................................................................12
MEANING OF THE AGREEMENT......................................................................................................................................................................................................................................12
A. PRINCIPLES OF INTERPRETATION......................................................................................................................................................................................................................12
INTEPRETATION OF A CONTRACT..............................................................................................................................................................................................................................12
SECTION 201: WHOSE MEANING PREVAILS..............................................................................................................................................................................................................13
SECTION 203: STANDARDS OF PREFERENCE IN INTERPRETATION............................................................................................................................................................................13
SECTION 204: SUPPLYING AN OMITTED ESSENTIAL TERM.......................................................................................................................................................................................13
SECTION 206: CONSTRUCTION AGAINST DRAFTER:..................................................................................................................................................................................................13
B. PAROL EVIDENCE......................................................................................................................................................................................................................................................13
SECTION 214: PAROL EVIDENCE RULE *ONLY CL.........................................................................................................................................................................................................13
STEPS FOR ANALYZING UNDER THE PAROL EVIDENCE RULE:........................................................................................................................................................................................13
EXAMPLE OF A MERGER CLAUSE:..................................................................................................................................................................................................................................14
EXAMPLE OF A NOM (NO ORAL MODIFICATION CLAUSE)..............................................................................................................................................................................................14
C. IMPLIED OBLIGATION OF GOOD FAITH; OTHER IMPLIED TERMS.........................................................................................................................................................14
SECTION 205: DUTY OF GOOD FAITH AND FAIR DEALING.............................................................................................................................................................................................14
2-306(2): EXCLUSIVE RELATIONSHIP (LUCY LADY DUFF CASE)...................................................................................................................................................................................15
2-309(3) NOTICE OF TERMINATION................................................................................................................................................................................................................................15
CONTRACT MODIFICATION.............................................................................................................................................................................................................................................15
PRE-EXISTING LEGAL DUTY...................................................................................................................................................................................................................................................15
EXCEPTIONS UNDER COMMON LAW:....................................................................................................................................................................................................................................15
UCC RULE REGARDING PRE-EXISTING LEGAL DUTY............................................................................................................................................................................................................15
WAYS TO GET AN EXCEPTION:..............................................................................................................................................................................................................................................15
CONSEQUENCES OF NON-PERFORMANCE..................................................................................................................................................................................................................15
A. EXPRESS CONDITIONS............................................................................................................................................................................................................................................15
EXPRESS CONDITION.......................................................................................................................................................................................................................................................15
2 TYPES OF EXPRESSED CONDITIONS..............................................................................................................................................................................................................................15
EXCEPTIONS TO EXPRESSED CONDITION.........................................................................................................................................................................................................................15
EXPRESS V. CONSTRUCTIVE CONDITIONS.......................................................................................................................................................................................................................16
B. BREACH........................................................................................................................................................................................................................................................................16
MATERIAL BREACH.........................................................................................................................................................................................................................................................16
PARTIAL BREACH............................................................................................................................................................................................................................................................16
TOTAL BREACH...............................................................................................................................................................................................................................................................16
SECTION 241: PARTIAL VS. MATERIAL BREACH.............................................................................................................................................................................................................16
SECTION 242: WHEN MATERIAL BREACH BECOMES TOTAL BREACH...........................................................................................................................................................................17
EXPECTATION DAMAGES..................................................................................................................................................................................................................................................17
COMPUTING EXPECTATION DAMAGES EXCEPTIONS:............................................................................................................................................................................................................17
ALTERNATIVES TO EXPECTATION DAMAGES...........................................................................................................................................................................................................17
A. RELIANCE AND RESTITUTIONARY DAMAGES...............................................................................................................................................................................................17
RELIANCE DAMAGES.......................................................................................................................................................................................................................................................17
RESTITUTIONARY DAMAGES...........................................................................................................................................................................................................................................17
B. SPECIFIC PERFORMANCE.....................................................................................................................................................................................................................................18
C. AGREED REMEDIES.................................................................................................................................................................................................................................................18
LIQUIDATED DAMAGES...................................................................................................................................................................................................................................................18
RIGHTS AND DUTIES OF THIRD PARTIES.....................................................................................................................................................................................................................18
INCIDENTAL BENEFICIARY:....................................................................................................................................................................................................................................................18
INTENDED BENEFICIARY:.......................................................................................................................................................................................................................................................18
DEFENSES..............................................................................................................................................................................................................................................................................18
ASSIGNMENT AND DELEGATION...........................................................................................................................................................................................................................................18

II. ESSAY..................................................................................................................................................18
AGENCY (ASK YOURSELF EVERY SECTION).....................................................................19
STATUTE OF FRAUDS: UCC......................................................................................................20
STATUTE OF FRAUDS: COMMON LAW................................................................................20
WARRANTIES...............................................................................................................................22
ANTICIPATORY REPUDIATION (AR).....................................................................................24
DEFENSES......................................................................................................................................25
DAMAGES.......................................................................................................................................29

I. Multiple Choice
TIPS FOR MC QUESTIONS FROM GRENARDO
First read call of the question and then scan responses
Look for absolute language, “Always, never, so long as, only if” as they sometimes indicate the answer is wrong
Eliminate definitely wrong options:
 Wrong if completely diff from other answers

 Wrong if applies the wrong rule, or the wrong part of rule, to the facts – Question 9(b)
 Wrong if misstates the rule
 Wrong if misstates the facts
 Wrong if gives you rule you’ve never seen
 Wrong but sounds right b/c it gives you a rule you have seen a lot but does not apply to fact pattern
GRENARDO KEY TAKEAWAYS
 Unilateral mistake can be a contract defense, if the other party should have known of the mistake.
 Unilateral contracts are very rare, except for reward offers. Most business contracts are bilateral.
 Courts do not reward bad behavior, so it is unlikely that one party will be allowed to knowingly take advantage of
another.
 Giving up a possible good-faith legal claim is giving up something of value.
 Substantial compliance is the standard applied to construction contracts, unlike the sale of goods where perfect tender
is required.
 Where a party to a service contract has substantially performed, he is entitled to be paid.
 If two parties agree to sell goods for less than market value, the courts will enforce that contract.
 Personal service Ks are non-delegable. There are some nuances to this rule, but this is the general rule. 

Practice Questions
Q1. On his 16th birthday, a young man purchased a new sports car from a dealership for $36,000. He entered into a written contract with the
dealership, which provided that he would make monthly payments of $1,000 for three years. The young man made the first 12 monthly
payments then failed to make any payments during the second year. The dealership threatened to sue on the balance, and, right after his 18th
birthday, the young man finally sent a letter offering to pay $10,000 of the remaining balance if he found a job in the next month. The next day
the young man was involved in an accident and the car was totaled. He remained unemployed and made no payments to the dealership. At the
time of the accident, the car was worth $18,000. If the dealership brings suit against the young man for breach of contract, what, if any, is the
plaintiff’s proper recovery?
Nothing. The one-sided power of avoidance is held by the infant only. An infant may disaffirm his or her contract at any
time prior to ratification. After reaching the age of majority, however, the infant may ratify or affirm the contract.
Q2. A patient entered into a written contract with a doctor to perform an outpatient surgery for $750. While performing the surgery, the doctor
noticed a suspicious growth on the patient, and asked whether the patient wanted him to remove this as well. The patient agreed orally to
have the growth removed for an additional $150, and the doctor performed both procedures. The patient did not remember the oral agreement
later, and paid the doctor $750 but refused to pay any more. The doctor sued the patient for the additional $150. The patient objected to the
introduction of testimony regarding any oral agreements during surgery. Which of the following considerations is most important to a
judicial ruling on the patient’s objection?
The proffered oral agreement regarding payment of the additional $150 occurred after the written agreement had
been signed. The parol evidence rule has never prevented proof of an oral or written agreement that varies or contradicts
the terms of a prior written contract.
Q3. On March 1, a homeowner and a painter entered into a written contract wherein the painter promised to paint the exterior of the homeowner’s
house for $3,000. Prior to the signing of the contract, the homeowner and the painter orally agreed that their contract would be null and void
unless the homeowner was able to obtain a $3,000 loan from National Bank before April 1. On March 31, the homeowner was informed by
National Bank that his loan application had been rejected. The next day, the homeowner telephoned the painter and informed him that the
deal was off. If the painter brings an action for breach of contract against the homeowner would the latter’s inability to secure the loan
provide him with a valid defense?
Yes, because the loan agreement was a condition precedent to the existence of the contract. Where the parties agree
that a condition precedent must occur before the contract is effective, it is generally agreed that the failure of the condition
to occur may be shown despite what otherwise would be deemed a total integration.
Q4. A man entered into a written contract with a nationally famous artist. Their contract provided that the artist would paint a full-color portrait of
the man, to be delivered by November l, in exchange for $15,000. A contractual provision recited that the artist “guarantees that the quality
of the portrait will be fully satisfactory.” The artist finished the portrait and presented it to the man on November 2. The man looked at the
portrait and said, “This is awful! I really expected better. I’m not paying for this!” The artist was enraged by the man’s reaction, as he felt the
finished portrait was one of his best works. Which argument is LEAST helpful to the artist in his efforts to recover on a breach of contract
theory?
(A) The man’s dissatisfaction with the quality of the portrait was not genuine.
(B) The man failed to make an adequate examination of the quality of the portrait.
(C) The man’s dissatisfaction w portrait was the result of delivery after 11/1, rather than of any lack of quality in the portrait.
(D) Because of a recent hand injury, the artist did not have sufficient time to produce a portrait of a quality
satisfactory to the man. Artist’s injury would not excuse his obligation to produce a quality portrait that was
satisfactory to the man. In contracts involving aesthetic taste, art, or personal judgment, the only performance that
will satisfy the condition of satisfaction is that the promisor be satisfied with the result.
Q5. A new restaurant, owned by a famous chef, hired a contractor to remodel the space several months prior to opening. As part of the remodel,
the contract stipulated that the contractor use KitchenCo ranges and ovens, renowned in the industry as high quality, in the kitchen space. The
contractor was unable to locate KitchenCo products, as the company routinely bad backorders of one year on its handcrafted equipment.
Rather than risk the opening date, the contractor located and installed ChefCo ranges and ovens; which were also well known in the industry
as a premium product, and similarly priced. When the chef saw ChefCo products in his kitchen, he immediately refused to pay the contactor,
demanding that KitchenCo products be provided.Which party is in breach?
(A) The contractor, because he materially breached the contract.
(B) The contractor, because he engaged in a unilateral modification.
(C) The chef, because the brand of high quality ranges is immaterial.
(D) The chef, because the contractor substantially performed the contract.
Q6. A landlord owns a building with a high, flat roof that overlooks the local ballpark. Part of the Little League World Series will be held at the
park. One of the landlord’s tenants has a younger brother who will play for their country in the series. The tenant tells the landlord about his
brother, and they agree that the tenant may rent the roof for the games where the tenant’s brother is playing. The tenant’s brother comes
down with a serious illness and is unable to travel to the series. The tenant asks to be excused from the roof lease, but the landlord refuses.
May the tenant be excused from the lease?
(A) Yes, because there was no writing so the agreement violates the statute of frauds.
(B) Yes, because the tenant’s purpose for renting the roof has been frustrated.When an event occurs that
dramatically reduces the value of the transaction to a party, frustration of purpose allows the party to be excused from
performance.
(C) No, because the games will still be held so the purpose of the lease is intact.
(D) No, because the tenant made a unilateral mistake and will not be excused.
Q7. A newly licensed attorney wants to open her own law office. Her aunt owns a commercial building that she uses for her own business, and that
also has a vacant office that the aunt would like to rent out. The aunt tells the attorney she can use the vacant office in her building without
charge if she helps the aunt with her legal issues, which would “be worth more than rent” to her. The attorney calls her aunt the next day to
firm up the arrangements for moving into the building. The aunt apologizes and tells the attorney that she found a paying tenant instead. The
attorney considers suing her aunt for breach of contract but wonders if there was consideration. Was the promise supported by consideration?
(D) Yes, because the agreement was a commercial transaction that would have benefitted the aunt.
Consideration can be distinguished from a condition on a gratuitous promise using three factors:
i) the language of the parties. Words suggesting benevolence rather than self-interest, such as” gift,” may indicate
a gratuitous promise that has a condition rather than consideration.
ii) commercial versus charitable or familial context. Bargains are standard operating procedure in the commercial
context, and gratuitous promises in that setting are rare. By contrast, gratuitous promises are made in the family
and charitable context more regularly.
iii) the benefit to the promisor. In case of doubt, the most important factor in determining whether a particular
performance is consideration or a mere condition on a gratuitous promise is whether the promisor benefits from
the promise.
Q8. A patron in an art gallery is shown a painting that the employee of the gallery says is an original work by a certain fashionable artist. The patron
has never heard of this artist, but buys the painting anyway, because he likes the colors and composition. A few weeks after the patron hangs the
painting, a visitor examines the painting and says it is not by the artist named by the gallery. The visitor, an expert in contemporary art, even says
that the painting bears no resemblance to that artist’s work. The patron sues the art gallery to void the purchase of the painting for fraud. Will the
patron win his suit?
No, because the patron did not rely on the false representation. A contract is voidable for fraud in the inducement only
if the party who seeks to void the contract reasonably relied on false statement.
Q9. A student needed to raise $1,200 to attend his graduation ceremony. The student had a comic book collection. Most of the comic books were
low value, but one was worth at least $300. The student notified neighbors and friends that he was selling his comic book collection and that
they could come to his home the following Saturday to view and buy. The following Saturday, his neighbor, Buyer, was the first to arrive.
Buyer had a grudge against the student. Buyer asked the student how much he wanted for the prized comic book. The student told Buyer that
a local shop had offered $225, but he wanted at least $300. Buyer offered the student $325 for the comic book if he would accept the offer
immediately and take it out of the sale. The student agreed. Buyer took the comic book with him and told the student to stop by Buyer’s
house for payment. When the student arrived for payment, Buyer handed the student the comic book and refused to pay. Student claims that
he had a contract with Buyer. How should the court decide?
There is a contract because the seller reasonably believed that the buyer had made a genuine offer, which he
accepted.
Whatever the buyer’s hidden intentions may have been, the seller reasonably believed that the buyer made an offer for the
comic book. The fact that the buyer offered an amount that was within the range of the comic book’s value and took it with
him serves to further reinforce that reasonable belief.
Q10. A pet store specializes in ball pythons. A man comes into the store and sees a particularly pale colored snake. He says he is interested in
breeding albino ball pythons, and wants to know if this particular snake is a female. The owner responds affirmatively. The man says that tile
snake will kick off his breeding program and purchases her. The man later looks up breeding techniques online and discovers that the snake
is not albino. He wants to return the snake for a full fund. Is the snake buyer entitled to a refund?
Yes, because his unilateral mistake was known by the shop owner. If a party to a transaction misunderstands a material
fact, that problem is not usually excused from performance. There is an exception if the other party has reason to
know or knew of the mistake.
Q11. When an investor purchased diamonds from a gem wholesaler, the two agreed to use ACE delivery. Unbeknownst to either, the investor
meant Acme Cargo Enterprises and the wholesaler meant American Carriers Express. The wholesaler then delivered the jewels to American, but
before they left the loading dock, American called the investor to verify his address. At that point, the investor discovered the mistake. Because of a
difference in insurance policies, there is a huge price difference between the two services, and the investor wished to rescind his agreement. Does
the investor have the right to cancel?
Yes, because there were two delivery services and, thus, a fatal ambiguity. In this instance, given the parties' lack of
genuine mutual understanding as to the delivery service they were agreeing to use, there was no "meeting of the minds."
Given this fatal ambiguity, no valid, enforceable contract existed, and as such, the investor is not obliged to perform.
Q12. The state government advertised for bids from general contractors on construction of a state office building. A contractor, who intended to
submit a bid, solicited bids from various sub-contractors for framing, electrical work, and plumbing. The contractor subsequently received a sub-bid
on the plumbing work for the project of $360,000. The next lowest competing sub-bid was $500,000. After compiling all the acceptable sub-bids,
the contractor submitted a bid on the total project for $5 million. This was the lowest bid submitted to the state, which accepted it on that basis. That
same day, the contractor notified the plumbing subbidder that his sub-bid was accepted. A week later, the subcontractor notified the contractor that a
transmission error had resulted in his plumbing sub-bid being too low by $200,000; the actual bid should have been $560,000 for the plumbing
work. The subcontractor had transmitted his sub-bid to the contractor via fax, and a problem with the line caused the bid to read "$360,000," instead
of the accurate "$560,000." If the subcontractor brings an action to rescind his contract with the contractor for the plumbing work, which of the
following is his strongest argument for rescission?
The contractor should have known that the sub-bid was erroneous, because it was $140,000 lower than any other
plumbing sub-bid. The tremendous disparity between the subcontractor's sub-bid in this question and the competing sub-
bids should have alerted the contractor to the fact that there was an error in the sub-bid, and his acceptance, in effect, took
knowing advantage of the subcontractor's unilateral error. Where the non-mistaken party knows or should have known of
the mistaken party's unilateral error, and takes advantage of it, the mistaken party is entitled to rescind.
Q13. An owner of an exclusive private summer camp for children hired a clown to perform for the children during the week of August 10 through
16. Unfortunately, the clown came down with acute laryngitis and informed the camp owner by letter that she would be unable to perform until
sometime in late August. On August 1, the camp owner sent the following letter by overnight mail to three other clowns. "I'm in a bind. I had lined
up a clown for the week of August 10 through 16, but she's sick and can't perform. I need another clown. You have to be here at camp by August 9
at the latest so I can prep you on this year's program. Camp is filled to capacity, so money is no problem." All three letters were identical and were
received by the respective clowns on August 2. As soon as the first clown received the letter, she cancelled her plans to perform at a local
renaissance fair the week of August 10, and without letting the camp owner know of her intentions, she showed up at camp during breakfast the
morning of August 9. After doing several cartwheels down the dining room center aisle, the first clown landed smiling in front of the camp owner
and sang: "Winter, spring, summer, or fall, all you have to do is call, and I'll be there. Your clown problems are over!" The camp owner took the
clown by the arm, and as inconspicuously as possible, slipped out of the dining room. She told the clown that the clown she had originally hired had
gotten her voice back and was going to perform as originally planned. The owner explained that because the first clown to receive the letter had not
communicated any response to the letter, the owner was not expecting her. Who will win if the first clown to receive the letter sues the summer
camp?
Camp, bc letter reasonably should have been understood as an offer that required a timely response in return.
Q14. An investor inherited a large, but minority block of a public company's stock. The public company's chief executive officer and major
shareholder, hired the investor as the public company's head purchasing agent and had the investor elected as a vice president of the public
company. On June 1 of last year, the investor invited bids from several coal companies for ten lots of metallurgical coal. A coal mining specialist
bid $750,000. This was an excellent price because it was substantially under the market price for metallurgical coal. The investor immediately made
a written agreement to purchase the coal from the coal mining specialist. Delivery was to be on or before August 10. The CEO was very pleased
with the deal that the investor had made and promised to have him elected to the public company's board of directors at the shareholders' meeting
scheduled for August 15. The investor had always wanted to be a director so that he could better protect his interest in the public company. Several
weeks before the coal was to be delivered, the coal mining specialist notified the investor that the coal mining specialist had made an arithmetical
error in pricing the coal and that the price should have been $1,000,000. The coal mining specialist requested a price increase to $825,000, which
the investor confirmed in writing after conferring with the CEO. When the investor received the coal mining specialist's bid for $750,000, the
investor was reasonably certain that coal mining specialist had made an error in its calculations.
Does the investor's acceptance of the bid result in an enforceable contract under the prevailing American rule?
No, because the minds of the parties had not met. This answer choice correctly captures the concept that there must be a
"meeting of the minds" before a court will grant relief in these types of circumstances. The coal mining specialist made a
unilateral mistake; although courts are sometimes reluctant to grant relief for unilateral mistakes, relief is almost always
granted when the other party is aware of the mistake.
Q15. A brother and sister grew up in the same modest home with similar educations, similar opportunities, and the affection of their parents.
However, as adults, their lives greatly differed. Although hardworking, the brother's family was poor and struggled with the basic necessities. In
contrast, the sister became a wealthy industrialist. Still, the brother and sister were close, so the sister eventually asked the brother and his family to
live in a guest house on her estate. After 25 years, the sister became concerned for her brother's future on the estate. Much of the sister's fortune had
already passed to her grown children, and she worried that if her children took over, they would evict her brother. So, the sister had her lawyer draft
a document containing the following agreement: "I hereby agree to purchase for my brother, in his name, the house specified in addendum number
one, said house being located downtown and not on my estate. In exchange, my brother agrees to make no claims against my estate after my death
or incapacitation." Both the brother and the sister signed and dated the document. Thereafter, the sister's children seized control of the family
empire, confined the sister to a sanatorium, and (as the sister's conservator) disavowed the agreement with the brother. The brother sued his sister's
children for breach of contract. The children asserted that the agreement was not enforceable because the brother's promise was not supported by
consideration. In most jurisdictions would the brother's agreement to make no claims against his sister's estate be regarded as sufficient
consideration for the sister's promise?
Yes, if the brother reasonably believed that he had a valid claim against his sister's estate. A promise is unenforceable
unless supported by consideration. This is the bargained-for exchange that is so essential to modern contract law. There
must be reciprocal inducement to enter the contract. Where a promise is made in exchange for another promise, the
requirement is usually met. Sometimes where a promise is illusory or based on a pre-existing duty, the contract will fail.
Q16. A shopping center engaged an asphalt company to redo its parking lot. The asphalt company promptly completed the project. However, the
shopping center subsequently discovered that the work had not been performed exactly according to the terms of the contract, although the mistake
was cosmetic and the parking lot was perfectly usable. Nevertheless, the shopping center sued the asphalt company, claiming that the parking lot
needed to be completely torn out and redone. The asphalt company defended on the ground that it had substantially performed its contractual duties.
How should judge rule?
The asphalt company will be required to pay only the diminishment in value of the parking lot as constructed,
because it substantially performed. The correct measure of damages is to place the injured party in the same position he
would be in had the breach not occurred. Damages are not intended to allow the injured party to profit or create a loss to
punish the other party. In contractor cases in which there has been substantial performance, the measure of damages is
typically the diminishment in value.
Q17. A neon sign designer contracted with a theater to create and mount a splashy new sign using the theater's new logo by June 1. After many
consultations, the sign was completed on May 30. However, due to a shortage of scaffolding, the sign was not mounted until June 5. The theater
director asserted that the theater owed nothing to the designer, because the designer had completed the project four days later than promised. Is the
theater required to render payment under the contract despite designer's delay in completing the project?
Yes, because the designer's breach was not a material one. While it is true that the designer breached the agreement, the
four-day delay would not constitute a material breach of the contract. In their agreement, the parties did not identify the
date of completion as a material term of the contract.
Q18. A struggling medical student from a small Midwestern town wished to buy some equipment to start up his own practice in his hometown once
he completed his residency. His uncle, a retailer of medical supplies, promised to sell the student some equipment at a discount when the student
was ready to set up his practice. The student had worked extra jobs in medical school and had managed to save $3,000. The uncle agreed to sell him
the equipment for $3,000. The actual retail value of the equipment was about $6,000. The student finished his residency and secured a building to
set up his practice. He sent a check to the uncle for $3,000 and awaited delivery of the equipment. A few days later, the uncle called and said that
the equipment had already been sold and that he was returning the student's check. The student was furious and threatened to sue the uncle. If the
student sues the uncle for breach of contract, who will prevail?
The student, because there was a bargained-for exchange. A bargained-for exchange exists when each party makes a
promise to perform in exchange for the other party's promise to perform
Q19. A college graduate in international relations was one of only five students to be selected for an internship with a well-known European think
tank. Although he spoke fluent German, the graduate was told that he was to work out of the Paris office. The graduate contacted his former
roommate, who had recently graduated with a degree in French literature, informed her that he was to leave for Paris at the end of December, and
asked her to teach him to communicate in French. The roommate was hesitant, knowing that it had taken her years to become fluent, but she agreed
to tutor the graduate three times a week for $200 per lesson. By December, it was clear that the graduate could not communicate well enough to
survive living in Paris. The graduate wishes to sue his former roommate for breach of contract. In defense, the roommate argues that the Statute of
Frauds operates to make the agreement unenforceable as an oral contract that by its terms cannot be performed within a year. Will she be
successful?
No, because the Statute of Frauds does not apply. SOF requires that certain contracts must be in writing and signed by
the party to be charged.
Q20. A schoolteacher was at the local shopping center to purchase some apparel. As the teacher entered a men's clothing store, horns began to
sound. The teacher was the one-thousandth customer to enter the store and had won a new suit. The teacher was presented with a store certificate
documenting that he had won and stating that he could claim his free suit. The form had a space for the identifying information, and one of the pre-
printed sections on the form indicated that the above-named individual was entitled to one suit of the color specified on the form. The teacher told
the salesperson that although the form specified that the teacher was entitled to a blue suit, the teacher would prefer a brown suit, if the store had
one available in his size. The salesperson agreed, and so the teacher filled in his name and other identifying information, signed the form and gave it
back to the salesperson. The salesperson took back the form and instructed the teacher to return next week for his suit. When the teacher returned
the following week, the teacher was presented with a blue suit in his size, even though the store had several brown suits in the teacher's size on
display. The teacher filed suit against the store and the salesperson for the brown suit. Is evidence of the teacher's conversation with the salesperson
admissible?
No, because the form the teacher signed indicated that the teacher was entitled to a blue suit. The parol evidence rule
prevents introduction of prior or contemporaneous evidence to contradict a fully integrated writing, such as the gratis award
agreement for the free blue suit.

Midterm Material to Know


Bilateral vs. Unilateral
 Bilateral: promise for a promise. It is binding on formation of the contract
 Unilateral: a promise for an act or forbearance. It is binding if and when the act/service is complete. It is a
condition precedent on performance .
o Unilateral contracts are very rare, except for reward offers. Most business contracts are bilateral
Option Contract
 Option K requires mutual assent and consideration. Consideration must be given which holds the offer open for a
reasonable amount of time and can be nominal (of little value) (hat, pen, $5, $10, etc., have to pay or hand over to be
valid)
o Section 87(2)
o 2-205 UCC- does not require consideration for an option K. time stated or a reasonable amount of time or 3
months and the only way it can be open for more than 3 months is if it is clearly stated
 Section 45: When a Unilateral K becomes Irrevocable
o When there is a unilateral offer, as soon as someone starts the performance the offeror cannot revoke the
offer
o The offeree has to complete performance as the offer stated or offeror doesn’t have to pay
o Offeror is bound, not offeree. The one doing performance can back out
 With unilateral K, an option K is created when performance begins
Section 27: Existence of K where written memorial is contemplated
o Manifestations of assent that are in themselves sufficient to conclude a K will not be prevented from so
operating by the fact that the parties also manifest an intention to prepare and adopt a written memorial thereof;
but the circumstances may show that the agreements are preliminary negotiations
When the parties are thinking about putting this into a formal K
 Even though we are thinking about putting this into a formal K we intend to be bound now
 Even though we are thinking about putting this into a formal K we are not intending to be bound until
the formal K is signed (Quake case)
Quake Factors:
Whether the type of agreement involved is one usually put into writing
Whether the agreement contains many or few details
Whether the agreement involves a large or small amount of money
Whether the agreement requires a formal writing for the full expression of the covenants
Whether the negotiations indicated that a formal written document was contemplated at the completion of the negotiations
Mailbox Rule Exception
Basis of Mailbox Rule: if an acceptance is sent by mail, it is valid regardless of if they ever receive it.
Exception: if you send a rejection, change your mind, and send an acceptation, then it depends on what gets there
first. If the rejection gets there first, there is a counter offer. If the acceptance gets there first, it’s a valid acceptance.
Contract Formation under UCC
UCC Article 2 
Article 2 applies to transactions in "goods", defined as "movable" property, like a car or computer. Does NOT apply to real
estate, services, or leases. Also does not apply to intellectual property. Does apply to consumers and merchants. 
UCC 1-103b
if there is a UCC provision it controls. If not, then common law 
UCC §2-201 + Merchant Exception
1. Writing sufficient to indicate a K for sale has been made between the parties
2. Signed by D or by D’s agent
3. Quantity of goods
a. Merchants* on both sides
b. Confirmation within a reasonable time
c. Sufficient against the sender (§131, §133, §134)
d. Has to actually be received
e. Party receiving has reason to know its contents
f. Objection is given within 10 days
 Contract under UCC is Enforceable if:
o Goods are specially manufactured for the buyer
o Not suitable for sale to others in the ordinary course of the seller’s business
o Seller has made a substantial beginning of manufacture or commitments for their procurements
 If D admits in his pleadings, testimony or otherwise in court that a K for sale was made
 Payment has been made and accepted or goods have been received and accepted—up to what you
paid
*someone who regularly deals in goods of a kind or holds herself out as having particular skills or knowledge involved in a
transaction” UCC §2-104(1)
MERCHANT* EXCEPTION:
 If you & your Buyer are both merchants*, and you sent him something in writing memorializing the oral
agreement (some courts consider detailed invoices sufficient), and he did not object, the oral contract is
enforceable.
UCC 2-204: Formation in General
1. A K for sale of goods may be made in any manner sufficient to show agreement, including conduct by both parties
which recognizes the existence of such a K 
2. An agreement sufficient to constitute a K for sale may be found even though the moment of its making is
undermined 
3. Even though one or more terms are left open a K for sale does not fail for indefiniteness if the parties have intended
to make a K and there is a reasonably certain basis for giving an appropriate remedy 
a. You can have OPEN terms, but still need the ESSENTIAL terms   
UCC 2-205: Firm Offer 
1. An offer by a merchant to buy or sell goods 
2. In a Signed writing 
3. By its terms, gives assurance that it will be held open 
4. Separately signed by the offeror 
 do not need consideration for an irrevocable offer 
 open for a reasonable amount of time (3 months), to reopen must renew or provide
consideration  
 *three month limit unless consideration is given or firm offer extended through a renewal.
UCC 2-206: Offer and Acceptance in Formation of K 
1. Unless otherwise unambiguously indicated by the language or circumstances  
a. An offer to make a K shall be construed as inviting acceptance in any manner and by any
medium reasonable in the circumstance 
UCC 2-305(1)(4):Open Price Term
(1) The parties if they so intend can conclude a contract for sale even though the price is not settled. In such a case the price is a reasonable price at
the time for delivery if (a) nothing is said as to price; or (b) the price is left to be agreed by the parties and they fail to agree; or(c) the price is to be
fixed in terms of some agreed market or other standard as set or recorded by a third person or agency and it is not so set or recorded.

If they intend to be bound w/o a price, then they can have an enforceable K
(4) Where, however, the parties intend not to be bound unless the price be fixed or agreed and it is not fixed or agreed there is no contract. In such a
case the buyer must return any goods already received or if unable so to do must pay their reasonable value at the time of delivery and the seller
must return any portion of the price paid on account.

If you intend to be bound only with a price, and you don’t have a price, then you wont have a K

Battle of the Forms


Predominate Purpose Test (Coakley)
 When there is a mixture of goods and services use the Predominate Purpose Test:
o How much are the services vs. how much are the parts.
o More value in the goods = UCC.
o More value in services = Common law.
 Coakley Factors
1. The language of the contract 
2. The nature of the business of the supplier 
3. The intrinsic worth of the materials 
Common Law: Mirror Image & Last Shot Rules
1. Mirror Image Rule: Terms of acceptance must be mirror image of offer.
2. Last Shot Rule: A party impliedly assented to counteroffer by conduct indicating lack of objection.
UCC 2-207: Additional Terms in Acceptance or Confirmation (Battle of the Forms Guide)
(1) Acceptance can be valid, even if it contains additional or different terms)
 Must be timely,
 Reasonable, and
 Not conditional upon new or different terms
(2) New term only controls IF:
 Both parties are merchants,
 New term doesn’t materially alter deal,
 Offer didn’t expressly limit acceptance to its terms,
 Offeror doesn’t object
(3) If the writings do not establish a contract the parties conduct may.
o Knockout rule: *Compare the writings and knockout what’s different*
 Looking at the forms, whatever is the same is contract. Whatever is different is "knocked out"
of the contract
 2-207 is different than common law = in common law if the terms are changed it is a counteroffer unless
accepted by performance.
UCC
Price quote = invitation to make offer
Purchase order = offer
Acknowledgment form = Acceptance
 
Electronic Contracting
Shrinkwrap Clickwrap Browsewrap
Warning on the outside informing Must scroll through the terms and Terms and conditions are located on
purchaser that the package contains click accept to proceed using the page. another page and there is not
terms and the purchasers use of the Must assent affirmatively to terms. requirement to read the terms before
product constitutes agreement to the use of the page.
terms. If dissatisfied with the Assent by performance. Not required
product/terms may return the product to accept terms before assenting.
within a number of days, failure to do Validity of browsewrap turns on
so purchaser is agreeing to the seller's whether the website puts a reasonably
terms   prudent user on inquiry notice of the
terms of the K.

 
Liability in the Absence of Bargained For Exchange
Ways to recover when there is no valid contract: 
A. Promissory Estoppel 
B. Restitution 
A. Promissory Estoppel
Section 90: Promissory Estoppel exists when (all 4):
1. A promise (lacking consideration) 
2. The promisor should reasonably expect the promise to induce action or forbearance on the part of the promisee
or third-party (From the perspective of the promisor. If I make this promise, should I reasonably expect my
promise to induce the other person to do something?) 
3. The promise does induce such action or forbearance  
4. Promise is binding if injustice can be avoided only by enforcement of the promise - remedy shall be limited as
justice requires  
Promissory estoppel is generally limited to only the costs incurred 
 Note: Promissory estoppel is missing consideration 
 Charitable subscriptions and Marriage settlements do not require a promise to induce such action or forbearance as
#3.
 Remedy is limited as justice requires it (costs and expenses incurred on reliance of promise)
o Look at other sources of income
o Everything within reason = expectation damages
 Illusory Promise: words in a promissory term but practically mean nothing = not consideration
 Contingent promise: “If this happens then you will get this”
Section 87(2): When Promissory Estoppel makes the offer Irrevocable
 Offer which the offeror should reasonably expect to induce action or forbearance of a substantial character on the
part of the offeree before acceptance and does induce such action or forbearance is binding as an option K to the
extent necessary to avoid injustice
i. Majority rule: have to provide the consideration for a binding K
ii. Sub/General K or Dilemma: Subs offer is what becomes irrevocable
 Promissory Estoppel Makes Offer Binding – this involves a very specific situation where a
subcontractor cannot revoke its offer/bid to a general contractor (GC) when the GC relies on the
sub’s bid to make its own bid to an owner and the owner accepts the GC’s bid; at this point, the sub’s
offer to do the bid is irrevocable based on the elements of promissory estoppel allowing the GC the
opportunity to either accept or reject the sub’s bid
 If the owner accepts G's bid, GC is bound to the owner to perform work at bid price
 Subcontractor can't revoke bid offer to GC after GC bid that includes sub bid is accepted by
owner, GC detrimentally relied on the subs offer.
B. Restitution
Promissory Restitution = services provided and then there is a promise to pay for it (Contract in reverse) 
Section 86: Promissory Restitution 
1. Promise made for benefit received is binding to the extent necessary to prevent injustice
2. A promise is not binding if:
a. The promisee conferred the benefit as a gift or promisor has not been unjustly enriched
b. To the extent that its value is disproportionate to the benefit
*on exam if you see a promise to pay double what services are worth that would be disproportionate in accordance with
§86(2)(b).*
c. Promissory Restitution is only not a contract because the promise came AFTER the act.
3. Elements of Promissory Restitution
a. A promise
b. Benefit previously received by promisor
c. Binding if necessary to prevent injustice
d. Exceptions: if the promisee conferred the benefit as a gift, or the promisor has not been unjustly enriched, or
to the extent that its value is disproportionate to the benefit
4. Doctrine of Unclean Hands  he who seeks equity must do equity. You must be acting in an equitable way to
recover. 
Restitution is missing mutual assent 
 Restitution - benefit received, doctor saved someone, expectation of being paid (missing MA) 
 Promissory restitution - services provided first, then someone promises to pay (contract in reverse order) 

 Statute of Frauds
Statute of Frauds = Contracts that must be in writing 
MY LEGS Mnemonic
Marriage Contracts
Year: contracts that cannot be performed within 1 year from the date of its making
Land contracts
Executor-administrator contracts
 The executor or administrator of a deceased person’s estate promises to answer for a duty of the decedent.
 Typically, this means that the executor or administrator promises that the deceased person’s debt will be paid.
Goods: sale of goods contracts for $500+
Suretyship contracts
 The promisor, also known as a surety or guarantor, promises to answer for the duty of another person.
 Like the executor-administrator contract, this typically means that the promisor promises to pay the other person’s
debt.
 Section 139: Promissory Estoppel (on essay too) / Enforcement by Virtue of Action in Reliance
Common Law ONLY
1. Elements of Promissory Estoppel:
a. A promise which
b. the promisor should reasonably expect to induce promisee’s action
c. which does induce the action or forbearance is
d. injustice can only be avoided through enforcement
2. Whether injustice can be avoided only by enforcement of the promise:
a. The availability and adequacy of other remedies, particularly cancellation and restitution
b. The definite and substantial character of the action or forbearance in relation to the remedy sought
c. The extent to which the action or forbearance corroborate evidence of the making and term of the promise,
or the making and terms are otherwise established by clear and convincing evidence
d. The reasonableness of the action or forbearance
e. The extent to which the action or forbearance was foreseeable by the promisor

Meaning of the Agreement 


 
A. Principles of Interpretation
Interpretation of a Contract
 Order of Preference
1. Express Terms--What does the language of the contract say?
2. Course of Performance--How the parties have performed on the contract at issue?
3. Course of Dealings--How they conducted themselves on previous contracts with each other? Does not
include negotiation
4. Trade Usage--How the term is understood in the industry?
5. Maxims (if applicable)--A traditional legal principle that has been frozen into a concise expression.
 Noscitur a sociis (The meaning of a word in a series is affected by others in the same series; or, a word may be
affected by its immediate context.
 Ejusdem generis (general term included with specific one will be deemed to include only things that are like
"includes the appliances, such as dishwasher and oven) 
 Expressio unius exclusio alterius (if a bunch of items are listed, it excludes anything not listed) 
 Ut magis valeat quam pereat (interpretation that makes the K valid is preferred to one that makes it invalid)
 Omnia praesumuntur contra proferentem (prefers the less favorable party/interpretation)
 Interpret contract as a whole
 Specific provision is exception to a general one 
 Handwritten or typed provisions control printed provisions 
 Public interest preferred  
 “Purpose of the Parties”
 Common sense 
Section 201: Whose Meaning Prevails
Innocent party's meaning prevails, typically
(1) Where the parties have attached the same meaning to a promise or agreement, both parties are bound to that
meaning  
(2) Where the parties have attached different meanings to a promise or agreement, party A’s meaning is attached if
party B knew of party A’s interpretation or had reason to know of this interpretation AND party A did not know or
did not have reason to know of party B’s interpretation
(3) Where both parties attached different meanings and neither has reason to know of the other’s meaning, then
there is no assent
Section 203: Standards of Preference in Interpretation
In the interpretation of a promise or agreement or a term thereof, the following standards of preference are generally
applicable:
 Interpretation which gives a reasonable, lawful, and effective meaning to all the terms is preferred to an
interpretation which eaves a part unreasonable, unlawful, or of no effect;
 Express terms are given greater weight than course of performance, course of dealing, and usage of trad, course of
performance is given greater weight than course of dealing or usage of trade, and course of dealing is given greater
weight than usage of trade;
 Specific terms and exact terms are given greater weight than general language;
 Separately negotiated/added terms are given greater weight than general standardized terms/other terms not
separately negotiated.
Section 204: Supplying an Omitted Essential Term 
When the parties to a bargain sufficiently defined to be a contract have not agreed with respect to a term which is essential
to a determination of their rights and duties, a term which is reasonable in the circumstances is supplied by the court. 
Section 206: Construction against Drafter: 
A contractual ambiguity should generally be resolved against the party who drafted the language in question  
Exception: a sophisticated party will usually outweigh the drafter restatement 
 B. Parol Evidence
The parol evidence rule states that once the parties have reduced their agreement to a writing, evidence of any prior oral or
written, or contemporaneous oral agreements is inadmissible to alter, vary, or contradict the terms of the writing. The parol
evidence rule has never prevented proof of an oral or written agreement that varies or contradicts the terms of a prior
written contract.
Section 214: Parol Evidence Rule *only CL
 Exclusionary rule of evidence
 4 corners of the contract- extrinsic evidence that is outside the contract
 Contract = fully intended final expression
 If they vary/contradict/supplement the contract - once you have a final expression of the promises
 Merger clause is conclusive - if not, is the agreement complete and specific = then it is a final expression and
fully integrated - if it’s fully integrated then excluding evidence
 If it is partial then it can be supplemented
Steps for analyzing under the Parol Evidence Rule: 
1. Does the agreement constitute a final expression of the parties? 
a. Is the contract complete and specific?
b. Merger/integration clause (This agreement represents the entire agreement of the parties and
supersedes all prior negotiations, agreements, and understandings between the parties) this is
conclusive evidence that this writing is the final expression of the parties 
c. If integrated only partially, then writing may not be contradicted, but the non-integrated part of the
agreement may be supplemented by such extrinsic evidence
1. Ex: if a term is left out, then you can use extrinsic evidence to supplement what was left
out, but cannot use extrinsic evidence to contradict a written term 
2. Is evidence being used to vary, contradict, or supplement the writing? 
a. If yes, then it is excluded 
3. If not, are there exceptions that allow for evidence to come in? 
a. If ambiguity exists, then evidence brought into explain the meaning of the agreement
1. Patent vs. Latent Ambiguity 
1. Patent - word itself is ambiguous
2. Latent - ambiguous upon provided evidence 
b. Formation Defect: Fraud, Duress, Mistake, Lack of Consideration
1. Fraud in the execution - says document is one thing but it actually is something else -
e.g. "sign this change of mortgage payments" but it is actually a deed 
2. Fraud in the inducement - lying about facts to get/induce someone to sign 
c. Reformation: when you revise the contract due to Scrivener's error (the person who writes the
contract writes it wrong). If this is the case, need clear and convincing evidence, higher burden than
the standard "preponderance of the evidence" 
d. Collateral Agreement: something that would…
1. Naturally be omitted from written agreement; or
2. Agreed for separate consideration 
e. Subsequent Agreement
1. If it was an additional agreement made after the signed contract, it is an exception that
may be included  
f. Conditions Precedent to Effectiveness
1. Condition precedent - If something doesn't happen, then the rest of the contract goes
away. Common with selling houses, buyer needs to qualify for the mortgage as a
condition precedent 
2. Sometimes these are oral, so then it is allowed as evidence in court 
Example of a Merger Clause: 
This agreement represents the entire agreement of the parties and supersedes all prior negotiations, agreements, and understandings
between the parties 
Example of a NOM (No oral modification clause) 
This agreement may not be amended, altered, modified, or changed in any way except in writing signed by all parties to this
agreement 

C. Implied Obligation of Good Faith; Other Implied Terms


 
Section 205: Duty of Good Faith and Fair Dealing
Every K imposes an obligation of good faith in its performance and enforcement so each party exercise their right to the
"fruits of the contract" 
Bad faith can be used for 3 situations:  
1. Permits the inclusion of terms and conditions which have not been expressly set forth in the written contract (Lucy Lady
Duff case) 
2. When there has been a bad faith performance, even though the express terms have not been breached 
3. Where one party has been granted discretion to do something under the contract by the express
 Where the contract involves matters of fancy taste or judgment, the promisor is the sole judge of his
satisfaction. If he asserts in good faith that he is not satisfied, there can be no inquiry into the reasonableness
of his attitude. Does not work if there is an express term that "we can choose not to use any of your movies"
(TSM/Waits case) 
Factors contributing to bad faith: parties possessing equal bargaining power, plaintiffs being financially vulnerable during
contract formation, or plaintiff's being represented by competent counsel 
Parol evidence rule usually doesn't apply as bad faith is implied 
2-306(2): Exclusive Relationship (Lucy Lady Duff Case) 
(2) A lawful agreement by either the seller or the buyer for exclusive dealing in the kind of goods concerned imposes unless
otherwise agreed an obligation by the seller to use best efforts to supply the goods and by the buyer to use best efforts to
promote their sale. 
2-309(3) Notice of Termination 
(3) Termination of a contract by one party except on the happening of an agreed event requires that reasonable notification
be received by the other party and an agreement dispensing with notification is invalid if its operation would be
unconscionable. 
Reasons for when reasonable notice would be needed 
1. Needing to sell the rest of your inventory 
2. Substantial unrecouped investment that is relying on the agreement (storage costs, advertising, employees) 
3. Need time to find a substitute arrangement  

Contract Modification
Pre-existing legal duty is not enough to modify a contract. It does not create consideration for a new contract.  
Exceptions under Common Law: 
 89(a) Unforeseen Circumstances - something comes up while performing the contract (drilling a hole and some
unexpected rock is discovered) so they get paid more because of the unforeseen circumstances, even though the
duty has not changed 
o Defense to that - mutual/unilateral mistake 
 89(c) Reliance on a Promise - Same example as above, hit the rock and then promise to pay extra but its not put in
writing and duties haven't changed. That is sufficient to pay them extra, even though the role hasn't changed. 
 Mutual Release – tear up old contract and make a new one; employee becomes more valuable, company and
employee agree to tear up old contract and write a new one to pay them more 
UCC Rule regarding pre-existing legal duty - Do NOT need consideration/additional consideration to modify a contract. 
Ways to get an exception: Need some type of defense (duress, fraud, etc) 
 
Consequences of Non-Performance
A.Express Conditions
 Express Condition
 Must be stated in unambiguous language because it literally must be performed or satisfied.
 Substantial performance will not suffice
 “if” “Unless” “Until”
 If it fails, then other parties’ duty is discharged
2 Types of Expressed Conditions
1. Condition Precedent: event that must exist/occur before a duty to perform will arise
2. Condition Subsequent: contemplates that a duty would be owed but subject to discharge on the happening of an
event after that duty had originally arisen
Exceptions to Expressed Condition
1. Waiver: intentional relinquishment of a known right
2. Estoppel: reliance on the other party.
3. Prevention: Conduct by one party to prevent the other party from fulfilling condition
4. Minor or Technical Condition: a condition that doesn’t affect the dealings.
5. Forfeiture: focused on the breaching party—suffers a great loss. Will D suffer a great loss?
o Exceptions to Forfeiture:
1. Sophisticated parties: Able counsel represented both parties. knowledge/experience or adequate
counsel
2. The parties included an Express Termination Clause in the contract.
3. The breaching party Maintained Ownership of Assets comprising the contract
Express v. Constructive Conditions
6. Constructive condition - one thing has to happen, and then the other thing will.  
7. Breach of constructive doesn't always breach contract like breach of express condition would. 
8. You build the house and I will pay you 

B. Breach
 When performance of a duty under a K is due, any non-performance is a breach
 Material Breach 
 If the obligee does not receive the substantial benefit of her bargain as a result of failure to perform or defective
performance, the breach is considered material.
 The nonbreaching party: may treat the contract as at an end
o i.e. any duty of counter performance owed by her will be discharged, and
o will have an immediate right to all remedies for breach of the entire contract, including total damages
Partial Breach 
 A breach of contract is partial if the obligee gains the substantial benefit of her bargain despite the obligor’s
defective performance.
 Examples would be insignificant delays in completing performance or small deficiencies in the quality or quantity
of performance when precision is not critical.
 The effect of a partial breach is to provide a remedy for the partial breach to the aggrieved party. The aggrieved
party is not relieved of her duty of performance under the contract
Total Breach 
 Non-breaching party completely discharged from contract and can sue for damages 
Section 241: Partial vs. Material Breach
In determining whether a failure to render or to offer performance is material, the following circumstances are significant: 
a. The extent to which the injured party will be deprived of the benefit which he reasonably expected;  
a. what they expected to get - what they actually received 
b. Big difference = Material. Small difference = partial 
b. The extent to which the injured party can be adequately compensated for the part of that benefit of which he will be
deprived;  
a. Is it hard to figure out how to compensate them? Material.  
b. Is it easy to figure out and we can compensate? Partial. 
c. The extent to which the party failing to perform or to offer to perform will suffer forfeiture; 
a. Forfeiture - When the breaching party suffers a great loss.  
b. Think of it as 'how easy is it to cure the issue at the beginning of contract' 
c. If there is a lot of forfeiture = partial breach. Not a lot of forfeiture = material breach 
d. If breach is late in the contract, then it is likely to be partial. If breach happens early, it is likely to be
material. (Jacob & Youngs Case) 
d. The likelihood that the party failing to perform or to offer to perform will cure his failure, taking account of all the
circumstances including any reasonable assurances; 
a. Are they likely to fix this mistake at all? If its likely they will fix it soon and do it right = partial. If likely
it will never be fixed = material. (Sackett case) 
e. Was the breaching parties conduct intentional/willful, negligent, or innocent 
a. Willful is more material 
b. Innocent is more partial  
c. Negligent could go either way 
Section 242: When Material Breach Becomes Total Breach 
A. See above 
B. The extent to which it reasonably appears to the injured party that delay may prevent or hinder him in making
reasonable or suitable arrangements 
C. The extent to which the agreement provides for performance without delay, but a material failure to perform or
to offer to perform on a stated day does not of itself discharge the other party's remaining duties unless the
circumstances, including the language of the agreement, indicate that performance or an offer to perform by that day
is important. 
a. Language of the contract has to indicate that the specific day mentioned was critical 
b. Says "time is of the essence, has to be done by this day" AND the circumstances also have to indicate
the day was important 
Expectation Damages
Computing Expectation Damages Exceptions:
Employee Breach
employer can recover cost of replacement + other losses (equivalent services at the lowest possible cost)
Contractor Breaches
a. Cost to complete; or
b. Dim. In value *Need all 4
1. Substantial performance; AND
2. Unreasonable economic waste; AND (defects in construction which are irremediable, or which may be
repaired without a substantial tearing down of the structure as in Jacob & Youngs)
3. Unintentional; AND
4. Trivial defect
Sale of Real Estate
(Crabby’s v. Hamilton) *What is the injured party going to do next?
a. Buyer breach = K price – Fair Market Value *buyer just sell to someone else
b. Seller breach = Fair Market Value – K price (or spec. perf.)
Alternatives to Expectation Damages
Alternatives to Expectation Damages: Reliance and Restitutionary Damages, Specific Performance, and Agreed Remedies
A. Reliance and Restitutionary Damages
Reliance Damages
Whether there were reliance damages?
 Profits lost due to a breach of contract are recoverable
 If it can be shown that full performance would have resulted in a net loss, the plaintiff cannot escape the
consequences of a bad bargain by falling back on his reliance interest.
Restitutionary Damages
Restitution = unjust enrichment = quantum merit
1. Reasonable value of the performance minus any payments already made; and
2. Recovery is undiminished by any loss which would have been incurred by complete performance
Full performance exception:
 If the nonbreaching party has fully performed his obligations under the contract and the breaching party’s only
remaining duty of performance is the payment of a liquidated or specified sum of money, the nonbreaching party
may not elect a restitutionary recovery but is limited to expectation damages.
B.Specific Performance
 Specific performance of a contract is a type of remedy that you would ask for when the other party breaches a
contract.
 When you ask for specific performance, you are asking that the court order the parties to proceed as planned under
the contract.
When damages would be inadequate
C. Agreed Remedies
Liquidated Damages
 Where a fixed or determinable sum of money has been specified in advance as the remedy for a particular type of
breach
(1) The damages must be difficult to ascertain or estimate at the time the contract was made.
(2) the amount agreed upon by the parties must be a reasonable forecast of the compensatory damages in the case of
breach
 Liquidated damages are not penalties, they are used in situations where potential damages from breach are hard to
calculate
o Puts a cash value $$$ on the damages
Remember the whole point of liquidated damages clauses that it we're not certain what the damages are going to be at one
point breaches as a result we're going to agree to it so if it's difficult to figure out what the damages are gonna be at the
time of Los that'll satisfy that element too

Rights and Duties of Third Parties


Incidental beneficiary:
 Does not typically have privilege to sue. The person not in privity.
 Promisor
 Promisee
Look at promisee’s intent to benefit 3rd party
Intended beneficiary:
 Has standing to sue. The people in privity.
 Two types of intended beneficiary: Creditor beneficiary and Donee beneficiary
Defenses
Third party right to sue is subject to any defenses the promisor might have to the K with the promisee.
Assignment and Delegation
Assignment Delegation
One party giving rights to another under a K. The duty to perform is given to another.
X and Y have a K. Y assigns her rights to Z. X and Y have a K. Y delegates her duties to Z.
Y = Assigner Y = Original Obligor & Delegator
Z = Assignee Z = New Obligor & Delegate
X = Obligor - obligated to perform X = Obligee – person receiving performance
(Delegator) is still on the hook until Z (Delegate) performs.
Unless there is a novation given by X (Obligee).
 Novation: clear manifestation by oblige to release
obligor
Delegation is acceptable unless:
(1) Personal service K that relies on skill, knowledge,
character, reputation, etc. of the individual; OR
(2) when K prohibits delegation
SECTION 317 SECTION 318

II. Essay
Read the call of the question first and follow it closely. It will tell you what to discuss and sometimes what not to discuss.

If you start running out of time on the essays, then start writing Issues, Rules, and Conclusions. 

Abbreviations

BROAD STEPS
ABBREVIATIONS
 Fraudulent 1. UCC Article 2, if the contract deals with the sale of goods. Goods are a tangible
misrepresentation: FM  movable item. OR
 Innocent 2. Common Law, if the contract deals with services, land, or something other than goods.
misrepresentation: IM  IS THERE A CONTRACT?
 Negligent A contract is a legal relationship between two or more parties with:
misrepresentation: NM  Offer → a manifestation of the intent to enter into a contract. Be sure that you distinguish a genuine offer
 Undue influence: UI  from an advertisement or an invitation to deal. Analyze whether there is an intent to contract.  Know the
 Implied warranty of difference between a UCC firm offer and the common law option contract.
fitness: IWOF  Acceptance → a manifestation of assent to the terms thereof made by the offeree in a manner invited or
 Implied warranty of required by the offer. Also, pay attention to whether the mode of acceptance is reasonable under the
merchantability: IWOM  circumstances, since the offeror can dictate a means of acceptance.
 Anticipatory Bilateral Contract: promise for a promise where the acceptance is a return promise.
Repudiation: AR  Unilateral Contact: promise for performance where the acceptance is the return performance.
This is another place where common law and the UCC differ; common law states there be a “mirror
 Loss in Value: LIV or LV
image,” where the UCC allows for battle of the forms.
 Other loss: OL 
Consideration → bargained for legal detriment. Are both parties doing something, or giving something,
 Cost avoided: CA  they are not legally obligated to do? In addition, be sure it was actually bargained for, and not in the past.
 Loss avoided: LA  Is there a consideration substitute? Such as promissory estoppel? This is synonymous with detrimental
 Statute of Frauds: SOF  reliance.
 Common Law: CL  DO ANY DEFENSES BAR THE CONTRACT?
 Uniform Commercial Lack of capacity → is one party a minor or otherwise lacking in capacity?
Code: UCC  Duress or coercion → has one party been forced into the contract in some way?
 Unilateral mistake: UM  Fraud → was the contract entered into under false pretenses?
 Frustration of Illegality → is the subject matter of the contract illegal? I.e., a contract to kill someone will not be
Purpose: FOP  enforced.
 Unconscionability: Uncon  Mutual mistake → if both parties are mistaken as to a basic assumption of fact that has a material effect
 Promissory estoppel: PE on the contract.
Unilateral mistake → if only one party is mistaken, this is generally not a defense. However, if the other
party knew or had reason to know of the mistake, it will be a defense to the formation of the contract.
Unconscionability → is the contract unconscionable? The court looks at a multitude of factors.
Statute of Frauds → does the contract need to comply with the statute of frauds? And if it needs to
comply, does it? Namely, is there a writing that is signed by the party to be charged? Or, in the
alternative, has there been substantial or full performance? MYLEGS
INTERPRET THE CONTRACT
Modification: Has the contract been modified in any way? Have there been any changes, either oral or
written, to the original agreement? Is there a preexisting duty owed by one of the parties?
Remember that under common law one needs consideration, while under the UCC consideration is not
required.
Parol Evidence: any prior or contemporaneous oral or written statements that vary or contradict a fully
integrated agreement cannot be brought in. However, if there is a partial integration, a prior or
contemporaneous oral or written statement can come in to supplement, so long as it doesn’t contradict. Be
wary of things that aren’t really parol evidence. Remember you can always bring in prior or
contemporaneous statements to prove that there was not a contract.
What if a term is AMBIGIOUS? Then we look to, in order:
Course of Performance → what has previously been done in this particular contract?
Course of Dealings → what has previously been done in prior contracts between these particular
parties?
Trade Usage: customary trade practices.
Is there an EXCUSE FOR NON-PERFORMANCE?
Express Condition Precedent: where the performance is conditional on the occurrence of some event
and the condition is stated in the agreement.
Implied Conditions: implied by law.
Precedent: must occur before the performance or before the performance is excused.
Concurrent: must occur at the same time performance is rendered.
Subsequent: if it occurs, it extinguishes the absolute duty to perform.
Impossibility: is the contract impossible for ANYONE to perform? Be wary, if someone somewhere can
perform, it is not truly an impossibility.
Frustration of Purpose: has the purpose of the contract been frustrated? For example, if you rented a
room to watch a parade, and the parade gets canceled, that would be frustration of purpose.
Impracticability: has the contract been made commercially impracticable by the occurrence of a
contingency the non-occurrence of which was a basic assumption on which the contract was made?
Essentially, did something that neither party could foresee happen to make the contract commercially
impracticable? Bear in mind that commercially impracticable is not the same as merely no longer
profitable.
Subsequent Agreement by the Parties: just like two consenting adults can contract for what they want,
they can also agree to contract OUT of something.
Was there an ANTICIPATORY REPUDIATION?
Is the Contract fully executory (has neither party performed)? If yes, there may be an anticipatory
repudiation. If no, anticipatory repudiation is not applicable and the party must give the other party time
to perform.
Did one party unequivocally state an inability or unwillingness to perform? If yes, then there may be
an anticipatory repudiation. If no, but grounds for insecurity exist, the other party may request adequate
assurances of performance. If such assurances are not received, it may be treated as an anticipatory
repudiation.
If there was an anticipatory repudiation, a party need not wait for the time for performance and may treat
the contract as breached and sue immediately for damages.
Any:  THIRD-PARTY BENEFICIARY, ASSIGNMENT OR DELEGATION ISSUES?
TPB: intended or not?
Delegation: a party can delegate duties
Assignment: a party can assign rights
Finally, WHAT IS THE REMEDY?
Expectation: what the non-breaching party would have had if the contract had been fully performed. This
is the most common type of damages.
Reliance: to put the non-breaching party in the position they would have been before the contract was
made.
Restitution: this is to prevent unjust enrichment and is used on things like promissory estoppel.
Consequential: things like lost profits, but it must be foreseeable.
Incidental: any damages that are “extra,” such as storage costs, costs to resell, etc.
Liquidated: where the contract specifies the amount of damages in the event of breach. This will be
upheld so long as it is reasonable.
Specific Performance: only used when goods are sufficiently unique and reg. damages will not suffice.
Agency
Principal = person with original authority
Agent = person who principal gives authority to
 In general, the death (or cessation, if the principal is not an individual) of the principal terminates the agent’s actual
authority. Under the Restatement’s approach, a termination of actual authority is only effective once the agent has
notice of the principal’s death.
 In general, a principal or agent may revoke most agency relationships at any time, unless any agency agreement
provides otherwise. Certain types of agency powers, such as those that are granted as securities and certain proxies,
are irrevocable. 
 A principal is bound for any unauthorized acts of his agent that the principal later ratifies. A principal ratifies an
agent’s act by manifesting assent that the act shall affect the principal’s legal relations or by conduct that justifies a
reasonable assumption that the principal so assents.
 When an agent holds herself out to a third party as possessing actual authority to act on behalf of a principal, the
agent impliedly warrants that she possesses the necessary authority. If the agent does not, in fact, have authority to
enter a contract on behalf of the principal, and the principal refuses to ratify the contract, then the third party may
seek damages against the agent for a breach of the implied warranty of authority. 
CHOOSE ¼: ONLY MOST APPLICABLE TO ESSAY QUESTION
Actual authority: principal telling agent to do something
Apparent authority: principal has said/done something to make a 3rd party believe the agent has authority to do the act in
question.
Inherent authority: agent has authority to do something by virtue of his/her position (IMPLIED)
Ratification: approval of act in question by the principle after the act occurs
Statute of Frauds: UCC
Whether UCC or CL governs?
R. UCC applies to the sale of tangible moveable goods. Common law applies to services and the sale of property.
A. UCC applies because this is a contract for the sale of tangible moveable goods (retail etc.)
C. This is governed by the UCC.
Does the contract fall within the Statute of Frauds?
A contract for the sale of goods worth $500 or more falls within the SOF.
Is the SOF satisfied? (one document. No tacking allowed!!!)
1. A writing satisfies the statute of frauds if it (1) is sufficient to indicate a contract of sale between the parties, (2) is signed
by or on behalf of the D, and (3) states a quantity.  Almost any writing will suffice. (i.e. unsent “gotcha” letter).
2. A signature may be any symbol made by the signer, such as initials, letterhead, emails, and texts.
Is there an exception to the SOF?
UCC Section 2-201(3)(a)(b), and/or (c)
Special manufacture of goods 
a) Specially manufactured for the buyer
b) It is not suitable for sale in the ordinary course of seller’s business 
c) Substantial manufacture has begun OR there has been a commitment to procure those materials (for the specific item)
Party whom enforcement is against admits that a K was made, but the K is not enforceable beyond the quantity of goods
admitted Partial payment or performance 
a) Made and accepted payment (Acceptance is a question of fact) 
b) Received and accepted the goods
AGENCY?
Do you need consideration to modify a contract under the UCC? NO
Statute of Frauds: Common Law
Whether UCC or CL governs?
R. UCC applies to the sale of tangible moveable goods. Common law applies to services and the sale of property.
A. Common Law applies because this is a contract for services or real estate.
C. This is governed by Common Law.
Does the contract fall within the Statute of Frauds?
K falls within SOF if:
1. K that cannot be completed within 1-year from the date of its making; OR
2. K for the sale of land.
A. 1. Plaintiff will argue that the services to be rendered cannot be completed within one year because…”___”
2. Plaintiff will argue that the (sale, lease, rent) of the land was within the SOF because…”___”
Does one document satisfy the Statute of Frauds?
A writing satisfies the statute of frauds if it
(1) is signed by or on behalf of the D, (A signature is really any symbol made by the signer such as initials, letterhead,
email, or text. Practically any type of writing)
(2) reasonably identifies the subject matter of the contract,
(3) is sufficient to indicate a contract has been made between the parties or offered by the defendant, and
(4) includes the essential terms
Does tacking allow the P to satisfy the SOF?
Tacking involves using (1) multiple documents (2) that
clearly relate to the same transaction, and (3)
 either (ONLY USE ¼ ON EACH EXAM Q)
o the signed document incorporates the unsigned documents by reference,
o the documents are physically together,
o all of the documents are signed by the D, or
o the D has acquiesced in the contents of the unsigned documents
See above @ signature.
See above for any document will suffice rule.
Does Promissory Estoppel apply? / Is there an exception to the statute of frauds?
Promissory Estoppel under the statute of frauds requires
1. A promise
2. the promisor should reasonably expect their promise to
induce action or forbearance on the part of the promisee or a third person;
3. the promise does induce such action or forbearance; and
4. the promise is enforceable notwithstanding the SOF if injustice can be avoided only by enforcement of the promise; the
remedy is limited as justice requires.
The factors relating to whether injustice can be avoided only by enforcement of the promise include:
a) the availability and adequacy of other remedies, particularly cancellation and restitution;
b) the definite and substantial character of the action or forbearance in relation to the remedy sought;
c) the extent to which the action or forbearance corroborates evidence of the making and terms of the promise, or the
making and terms of the promise are otherwise established by clear and convincing evidence;
 See above.
d) the reasonableness of the action or forbearance; and
 See above.
e) the extent to which the action or forbearance was foreseeable by the promisor.
 See above.
Analysis.
1. P will argue there was a promise
2. P will argue the promisor reasonably expected to induces P’s action
3. P will argue the promise did induce P’s action
4. P will argue that enforcement is the only way to avoid [what would the injustice be?]
Factors to analyze (best to do all 5)
o The availability and adequacy of other remedies, particularly cancellation and restitution
o The definite and substantial character of the action or forbearance in relation to the remedy sought
o The extent to which the action or forbearance corroborate evidence of the making and term of the promise, or
the making and terms are otherwise established by clear and convincing evidence
o The reasonableness of the action or forbearance
o The extent to which the action or forbearance was foreseeable by the promisor
AGENCY?
Does there need to be consideration to modify a contract under the CL? YES
Warranties
(look to CoQ; will tell you issues)
Whether UCC or CL governs?
R. UCC applies to the sale of tangible moveable goods. Common law applies to services and the sale of property.
A. UCC applies because this is a contract for the sale of tangible moveable goods (retail etc.) Common Law applies
because this is a contract for services or real estate.
C. This is (_______)
Were there any expressed warranties, and were they valid?/Whether there is an express warranty?
Expressed warranties can be written or oral and are expressed by 3 ways: 
1. Making a representation of the goods
2. Giving a description; OR 
3. Displaying a sample model 
Mere puffery, sales talk, or the seller's opinion does not serve as a binding commitment; affirmation of fact must be
objective and capable of being proven true or false. 
For example, car is “best in class” or “superb” equals puffery but car is “fastest in class” could be verified. Similarly, a
seller’s opinion or commendation does not typically create an express warranty. For instance, “Ford trucks are the best”
or “you’re going to love this boat” do not amount to express warranties.
Whether there was an implied warranty of merchantability (IWOM)?
An IWOM requires:
1. Merchant seller – a party who regularly deals in goods of the kind or holds itself out as having particular knowledge
about the kind of goods here
2. Whether goods are fit for the ordinary purpose for which such goods are used,
3. Must establish standard of merchantability in the trade to determine whether goods would pass without objection in
the trade
Whether there is an implied warranty of fitness for a particular purpose?
(1) Seller has reason to know any particular purpose for which the
goods are required and
(2) Buyer is relying on the seller’s skill or judgment to select or
furnish suitable goods
(3) Not limited to merchant sellers
(4) Breach does not require goods are defective in any way, just that
goods are not fit for the buyer’s particular purpose
E.g., buy shoes, ordinary purpose walking but buy for mountain
climbing; tires, ordinary purpose on highway and streets but buy
for off-roading
Ex. One buys shoes, their ordinary purpose is walking but they buy them for mountain biking.
Whether there was a valid disclaimer?/ Whether there is a disclaiming warranty?
Disclaimer = tries to get rid of the warranty
Express warranty prevails if the disclaimer & warranty are not consistent. 
1. IWOM disclaimer: 
Can be oral or written;
If written must be conspicuous; 
Conspicuous: easy to see, blatantly obvious
Must mention merchantability.
2. IWOF disclaimer:
Must be in writing, 
Conspicuous; 
See above for Merchantability.
General language is okay.
3. “as is” or “with all faults” disclaimers work for IWOM and IWOF
Gets rid of both IWOM and IWOF; is an exception
AGENCY?

Anticipatory Repudiation (AR)


(ONLY house CoQ; same as in class AR hypothetical)
 Anticipatory repudiation occurs when a party states an intention not to perform unless conditions outside of the contract are met.
 A request to modify the contract, including the price term, does not constitute anticipatory repudiation.
o Whether anticipatory repudiation occurs is a question of fact.
 Anticipatory repudiation may be rescinded if the repudiation is retracted and notification of such retraction is given (1) before the
other party materially relies on the repudiation or (2) indicates that he deems the repudiation to be final.
o Element (2) may be established by notifying the repudiating party, bringing suit, or in any other way that communicates an
election to treat the contract as repudiated.
Whether UCC or CL governs?
R. UCC applies to the sale of tangible moveable goods. Common law applies to services and the sale of property.
A. UCC applies because this is a contract for the sale of tangible moveable goods (retail etc.) Common Law applies
because this is a contract for services or real estate.
C. This is (_______)
Whether there is anticipated repudiation?
(1) Anticipatory repudiation occurs by words or conduct and
(2) requires a clear manifestation of intent not to perform the contract on the date of performance
Assuming there was anticipatory repudiation, what are the non-breaching party's three options, and which will they
likely choose?
The three options of the non-repudiating party after anticipatory repudiation by the other party:
1. Treat the K as rescinded or terminated;
2. Treat repudiation as a total breach and sue for expectation damages; or
3. Ignore the repudiation, urge performance, wait for the specified time of performance, and then sue if the repudiating
party does not perform.
Assuming there was anticipatory repudiation, was there retraction?
A party can take back its anticipatory repudiation by retracting it.
A retraction is effective if it comes to the attention of the non-breaching party before the non-breaching party:
1. Materially changes position (doesn't have to give notice); or
2. Indicates to the other party that it considers the repudiation to be final (does have to give notice).
Assuming there was not anticipatory repudiation, were there reasonable grounds for insecurity?
Whether there are reasonable grounds for a party to believe that the other party will not perform its part of the contract
when the time comes due is a fact question based on commercial reasonableness.
1. Buyer’s exact words or actions
2. Course of dealing or performance
3. Nature of the sales contract and the industry

Assuming there were reasonable grounds for insecurity, [either] was the adequate assurance requested proper [or]
what could the non-breaching party properly request as adequate assurance?
When there are reasonable grounds for insecurity, the non-breaching party can demand adequate assurance from the other
party. What constitutes proper adequate assurance is a fact question based on commercial reasonableness, and a demand for
adequate assurance is not proper if it changes the substance of the K. Failure to provide adequate assurance constitutes
anticipatory repudiation and a total breach of the contract.
1. Does this change the substance of the contract?
2. What other adequate assurance could be proper?
AGENCY?
Defenses
When it comes to issue-spotting of any kind, if one element or factor of a rule is present, then you've spotted an issue so make that concept
into an IRAC. For example, even if a defense would not succeed, but one of its elements or factors is met, then you would add an IRAC for
that defense. 

Whether UCC or CL governs?


R. UCC applies to the sale of tangible moveable goods. Common law applies to services and the sale of property.
A. UCC applies because this is a contract for the sale of tangible moveable goods (retail etc.) Common Law applies
because this is a contract for services or real estate.
C. This is (_______)
Whether the minor can disaffirm the contract?
1. Absent some overreaching or fraud, and assuming the contract is reasonable and fair, a disaffirming minor must make
restitution for either the benefit received under the contract or the depreciation in the value of the property. 
2. The minor is always liable for necessities. 
3. Minor ratifies the contract after he reaches the age of majority if he does not disaffirm the contract within a reasonable
time.
A. 1. P will argue there was no tortious conduct (if applicable) and therefore should be able to disaffirm the contract.
2. If applicable
3. if applicable
Whether there was incapacity?
Only pick cognitive or volitional; only have to write applicable rule
1. A contract is waivable if the offeree was unable to understand (cognitive) the nature and consequences of the
transaction; OR
2. The offeror has constructive knowledge that the offeree is unable to act (volitional) in a reasonable manner in relation to
the transaction; UNLESS 
3. The contract is fair, or it would be unjust to void. 
Whether the offer is voidable due to intoxication?
A person incurs only voidable contractual duties by entering into a transaction if the other party has reason to know that by
reason of intoxication:
He is unable to understand in a reasonable manner the nature and consequences of the transaction (cognitive); OR
He is unable to act in a reasonable manner in relation to the transaction (volitional).

BELOW: IF GET ONE, IRAC ALL THREE


“so and so was SHOCKED”: undue influence, procedural, substantive!!!
Whether there was undue influence?
One party takes advantage of another’s weakness of mind or there is a disparity in bargaining power. Shown by:
1. Discussion of the transaction at an unusual or inappropriate time
2. Consummation of the transaction in an unusual place
3. Insistent demand that the business be finished at once
4. Extreme emphasis on untoward consequences of delay
5. The use of multiple persuaders by the dominant side against a single servient party
6. Absence of 3rd party advisers to the servient party
7. Statements that there is no time to consult financial advisers or attorneys.
A. 1. Plaintiff will argue that the discussion was inappropriate because…
Etc. through 7 factors
Whether the contract was unconscionable? / Whether there was Unconscionability?
Absence of one party’s meaningful choice with terms that unreasonably favor the other party.
Procedural Unconscionability:
1. Setting of the transaction 
2. Experience and education of the party claiming unconscionability 
3. Whether the contract contained fine print 
4. Whether the seller used high pressure tactics 
5. Any disparity in the parties’ bargaining power 
Substantive Unconscionability: 
6. Terms of contract are unfairly one sided 
Some jurisdictions require only procedural or substantive unconscionability, while some jurisdictions require both; those
that require both may use a sliding scale, meaning if you have a lot of unconscionability for procedural, then you do not
need as much for substantive (and vice versa)
A. 1. See above @undue influence
2. Plaintiff will argue they lacked the education and experience to understand the terms of the contract (ex: GED education
v. J.D.)
3. Plaintiff will argue that important terms were hidden in fine print maze
4. See above @undue influence
5. See above @undue influence
Whether there was economic duress?
R. 1. Wrongful or improper act or threat by defendant
2. Wrongful act or threat by defendant causes the plaintiff to involuntarily accept the terms of the defendant
3. plaintiff has a lack of reasonable alternatives other than acceptance
A. Analyze all three elements in addition to Unconscionability and undue influence
Whether there was fraudulent misrepresentation?
1. D made statements inconsistent with the facts
2. (a) D knew it was false OR was unsure if it was true; and (b) intended to mislead P
(scienter = mental state consisting of an intent to deceive, manipulate, or defraud);
3. The misrepresentation was material 
4. P reasonably relied on and was induced by the misrepresentation 
5. The misrepresentation caused P damage.
Whether the misrepresentation was negligent?
1. See above @ 1, 3, 4, 5
2. D would have known it was false through reasonable care.
A. 1. See above @ 1
2. P will argue that D should have used reasonable care by [...] to ensure the information that he represented was true.
3. See above @ 3
4. See above @ 4
5. See above @ 5
Whether the misrepresentation was innocent?
See above @ 1, 3, 4, 5.
A. 1. See above @ 1
2. Does not apply
3. See above @ 3.
4. See above @ 4.
5. See above @ 5.
Whether non-disclosure is equivalent to an assertion?
Non-disclosure of known facts = assertion of non-existence if known that disclosure would:
1. Prevent previous assertion from being a fraudulent or material misrepresentation
2. (a) correct mistake of basic assumption on which the contract is being formed AND (b) such ND is a
failure to act in good faith and in accordance with reasonable standards of fair dealings.
3. Correct mistake of content or effects of the writings;
A. Analyze all four elements
Whether this contract violates public policy?
Public Policy: The provision or contract violates public policy if it’s contrary to laws, statute, constitution, judicial decision
or mores of society
Covenant not to compete: Not to compete w previous employer within reason on geography, scope, or duration
Blue Pencil Policy: Line through something to make it more reasonable
If the K is
1. Contrary to law, judicial opinion; OR
2. Mores of society
Whether there was a Mutual Mistake?
1. Mutual mistake is a belief not in accordance with the facts at the time the contract was made by both parties
2. Belief has to be relating to a basic assumption on which the contract was made
3. Has a material effect on the agreed exchange of performance
Material effect= if the party knew about it they would have either not entered into the contract or it would have drastically
changed the amount paid 
Whether there was Unilateral Mistake?
1. “See above @ Mutual Mistake”
2. Effect of the mistake such that enforcement would be unconscionable; or 
Unconscionable: severe enough to cause substantial loss 
3. Other party had reason to know of mistake or his/her fault caused the mistake
A. 1. See above @ mutual mistake.
2– 3. Analyze individually
Who bears the risk?
A party may bear the risk based on the following:
a. Agreement of the parties
b. One party is aware, at the time the K is made, that it has only limited knowledge with respect to the facts to which the
mistake relates but treats his limited knowledge as sufficient;
c. The court allocating risk to one party based on what is reasonable under the circumstances
A. 1. If applicable
2. If applicable
3. Always.
Whether there was Impossibility?
1. The person we need has died
2. The thing/good we need are destroyed
3. Because of a law or judicial order what we are trying to do is illegal
Whether there was Frustration of Purpose?
1. The purpose frustrated by the supervening event must have been the “principle purpose” of the party making the K;
2. The frustration must be substantial 
3. The non-occurrence of the frustrating event must have been a “basic assumption” of the K 
4. The frustrating event must occur without the “fault” of the party seeking discharge; and
5. The party seeking relief does not bear the risk of that frustrating event under either the language of the K or the
surrounding circumstances
Whether there was Impracticability?
1. Performance is made commercially impracticable as the unforeseen supervening event dramatically reduces the value of
the K
2. The increase in the cost of performance would be far beyond what either party anticipated 
3. See above @ (3)
4. See above @ (4)
5. See above @ (5)
AGENCY?
Damages
Whether UCC or CL governs?
R. UCC applies to the sale of tangible moveable goods. Common law applies to services and the sale of property.
A. UCC applies because this is a contract for the sale of tangible moveable goods (retail etc.) Common Law applies
because this is a contract for services or real estate.
C. This is (_______)
What is the loss in value (LIV)?
LIV is what a party expected to receive minus what that party actually received
What is the other loss?
“Other loss” includes incidental damages, and consequential damages: 
1. Incidental damages include additional costs incurred after the breach in a reasonable attempt to avoid loss, even if the
attempt is unsuccessful.
2. Consequential damages include such items as injury to person or property caused by the breach, as well as loss of profits
on a collateral contract that are foreseeable OR reasonably certain.
What was the cost avoided?
The cost that a party expected to incur to complete the contract – the cost actually incurred
Costs expected – costs actually incurred
1. When the injured party terminates and claims damages for a total breach, the breach may have beneficial effect on the
party by saving it the further expenditure that would otherwise have occurred.
Was there loss avoided?
When the injured party terminates performance and claims damages for total breach, the breach may have a further
beneficial effect on that party by allowing it to avoid some loss by salvaging and reallocating some or all of the resources
that otherwise it would have had to devote to the performance of the contract.
Whether there was a duty to mitigate? (loss avoided)
1. Damages are not recoverable for the loss that the injured party could have avoided without undue risk, burden or
humiliation
2. Injured party is not precluded from recovery by the rule states in subsection (1) to the extent that he has made reasonable
but unsuccessful efforts to avoid loss.
Whether an employee had a duty to mitigate damages from wrongful termination by an employer?
When an employee is wrongfully discharged, (1) they have to try to find suitable employment, (2) substantially equivalent
to the position lost, (3) suitable to background and experience.
Whether the damages were foreseeable? (loss in value)
Damages are contemplated at the time K was made with a focus on breaching party
2. Losses for which D had reason to know (constructive knowledge).
3. Loss must be foreseeable as a probable result of the breach (proximate cause). (Does not extend to remote losses).
Whether damages were Reasonable certainty? (loss in value)
Damages have to be reasonably certain in: 
a. Nature – can quantify the damages 
b. Origin – came from the breach 
Damages cannot be speculative
What is the general measure in damages? SHOULD BE LAST IRAC FOR DAMAGES!!!
loss in value + other loss – cost avoided – loss avoided
avoid putting the same number in multiple places

Computing Expectation Damages Practice Problems


1. Owner hires builder to construct a building for a total price of $200,000. The estimated total cost of construction is $180,000. The owner
breaches by unjustifiably terminating the contract when the work is partly done. At the time of termination , the owner has paid the builder
$70,000 for work done, and the builder has spent a total of $95,000 for labor and materials (some of which are incorporated in the partially
completed building). After the owner’s breach the builder is able to resell $10,000 of materials purchased for the project.
 Loss in value: $200,000-$70,000 = $130,000
 Other loss: $0
 Cost Avoided: $180,000-$95,000=$85,000
 Loss Avoided: $10,000
$130,000 + $0 − $85,000 − $10,000 = $35,000
2. Employer hires employee under a two-year employment contract for a salary of $50,000 per year, payable in installments at the end of each
month. Six months after the employee starts work, the employer wrongfully discharges her. The employee looks for work for three months,
but is unable to find a job. Finally, she hires an employment agency, paying it a fee of $1,000. Three months later she obtains a job (similar
to one from which she was fired) paying $45,000 per year.
Loss in value: ($50,000x2)-($50,000/2) = $100,000-$25,000 = $75,000

 Other loss: Incidental (-$1,000)
 Cost Avoided: $0
 Loss Avoided: $45,000
$75,000 + (-$1,000) − $0 − $45,000 = $31,000
AGENCY????

You might also like